Thermodynamics (statistical): Boltzmann distribution derivation

  Рет қаралды 34,112

lseinjr1

lseinjr1

Күн бұрын

Пікірлер: 68
@lseinjr1
@lseinjr1 5 жыл бұрын
(for *mobile* users) 0:14 Introduction 0:30 Canonical ensemble 3:44 Two constraints 5:21 Number of ways to achieve distribution (Ω) 8:03 Stirling's approximation 9:36 ln Ω 14:34 Lagrange's method of undetermined multipliers 16:15 Derivative of ln Ω 18:02 Apply Stirling's approximation twice 20:53 Derivative of N ln N 27:14 Full derivative equations 30:36 Use of definition of N 33:20 Canonical ensemble partition function Q 35:08 Boltzmann distribution
@nvrhanna
@nvrhanna 4 жыл бұрын
Thanks!
@veronicanoordzee6440
@veronicanoordzee6440 5 жыл бұрын
After 11:08: it should be the summation of ln(n,!), not just of n,! It's not so easy to copy something you've already written down ;-)
@aww2historian
@aww2historian 5 жыл бұрын
Yes please fix! Got super confused after that
@Asphalt9_talks
@Asphalt9_talks 11 ай бұрын
I had fun watching this. Really helped me understand entropy from this video. Helped me get over a lot of frustration in understanding where did entropy come from. Thanks a lot.
@lseinjr1
@lseinjr1 11 ай бұрын
Glad it helped!
@matteopiccioni196
@matteopiccioni196 3 жыл бұрын
very good derivation, i followed it up the end, and i got it a lot. Yes at 11:30 there is a mistake, i would say only an oversight, because then proceeding the derivation you correct yourself.
@guest_of_randomness
@guest_of_randomness 6 ай бұрын
it's always the random video from youtube that save my day! thank you so much, you are a great explainer !
@londonmackerel7462
@londonmackerel7462 5 жыл бұрын
I have doubts about minutes 21 to 22 of this video. Here N is differentiated with respect to nk. However, N is a constant (isn't it?) so its derivative should be zero. That is, even if an nk might vary, N will not as the other n terms vary to keep N constant. It seems to me that If you treat N as a constant and so with a derivative of zero, you derive the Boltzman distribution more easily.
@GoogleAccount-mz6vn
@GoogleAccount-mz6vn 3 жыл бұрын
I think we didn't consider the constraint because at that point we have had already imposed the restriction by deciding to use the extra terms (in the final derivative) presented by Lagrange's Method of Undetermined Multipliers. We shouldn't have imposed it twice since the substractions of the second and third terms of the final derivative already was there to account for it. chem.libretexts.org/Bookshelves/Physical_and_Theoretical_Chemistry_Textbook_Maps/Book%3A_Thermodynamics_and_Chemical_Equilibrium_(Ellgen)/21%3A_The_Boltzmann_Distribution_Function/21.02%3A_Lagrange's_Method_of_Undetermined_Multipliers
@tedsheridan8725
@tedsheridan8725 2 жыл бұрын
With Lagrange optimization, the constraints are considered elsewhere. If F = x+y+z, dF/dx = 1 regardless of the constraint that x+y+z = 7
@tanhevg
@tanhevg 2 жыл бұрын
Thanks for making this video. This is really helpful.
@lseinjr1
@lseinjr1 2 жыл бұрын
Glad it was helpful!
@LibertyAzad
@LibertyAzad Жыл бұрын
Took me a while to find such a clear explanation. But I still wonder about the thermodynamic vs. empirical temperature. I guess at the end you were referring to thermodynamic temperature. Thank you for your videos!!!
@pavelmorales5491
@pavelmorales5491 3 жыл бұрын
Thank you, a good detailed explanation! PS: In 25:09 you missed a one but did not affected further calculation.
@purielmafiacity6193
@purielmafiacity6193 Жыл бұрын
Essa aula é muito boa, agradeço por sua explicação. 🎉
@SlowRiderDucati
@SlowRiderDucati 3 жыл бұрын
Hey Dude, Thanks it finally makes sense!
@tedsheridan8725
@tedsheridan8725 2 жыл бұрын
I've been watching several related videos, this is the first one that explained it clearly. Thanks! One thing I'm still confused on - the discrete nature of energy levels of the various systems. Is energy discrete here because of quantized energy levels in the QM sense? Or are we taking continuous energy and putting them into discrete bins of arbitrary width?
@lseinjr1
@lseinjr1 2 жыл бұрын
Great question! No assumption is made that the energy levels of individual systems are discrete. Even in QM, energy is not necessarily quantized - an electron has quantized energy levels when it is "bound" by the nucleus, but not when it is unbound. It is true that there are a limited number of energy levels available for the N systems in the Boltzmann distribution, but that is because there are only a finite number (N) systems involved. The total energy of the system is the sum of the energies of the N individual systems. Does that answer your question?
@paulg444
@paulg444 4 жыл бұрын
Great video , would be nice to do a follow on where momentum and position are accounted for and the sum of momenta of each microstate is conserved.
@jianqiuzhang4607
@jianqiuzhang4607 5 жыл бұрын
Really appreciate you. This helps me a lot!
@AlphaNumeric123
@AlphaNumeric123 4 жыл бұрын
Excellent content presentation. The best resource I've found on this topic. Thank you so much.
@lorianamesia331
@lorianamesia331 4 жыл бұрын
Nice picture. I love the painter who did it
@100rubmighty
@100rubmighty 4 жыл бұрын
Your Lagrange multiplier method is incorrect: (1) The constraint is \sum n_j = N, the Lagrange multiplier method requires \alpha*(\sum n_j - N). The multiplier is applied to the constraint. Similarly for E_tot (2) You would have realized this when you supplement the equations with partial derivatives w.r.t. \alpha and \beta. These are supposed to enforce the constraints. (3) Fortunately for you, when you take the derivative w.r.t. n, the N and E_tot terms drop out. But the mistake checks itself out only for the purpose of the video. Any other conclusions will be wrong.
@lseinjr1
@lseinjr1 4 жыл бұрын
No. 1. Since Σ n_j = N, dN = 0 = Σ n_j dn_j, so α Σ n_j dn_j = 0 as well. Similarly, Σ n_j E_j = E_total, so dE = Σ n_j E_j dn_j = 0 as well, so β Σ n_j E_j dn_j = 0. Do you have a citation for your method?
@Sky-pg6xy
@Sky-pg6xy 3 жыл бұрын
Where did the natural log go when he wrote the n’s as a summation?
@lseinjr1
@lseinjr1 3 жыл бұрын
See the *NOTE* in the description box.
@williamwolfe8708
@williamwolfe8708 4 жыл бұрын
Thanks for the content. Was interesting to see all the energy spent in changing colors. At this level of abstraction, and expected education level of the viewer, the probability the colors were helpful was approximately 0. Just kidding around -- enjoyed the nice video, thanks!
@NicolasSchmidMusic
@NicolasSchmidMusic 4 жыл бұрын
noooo colors always help :)
@tedsheridan8725
@tedsheridan8725 2 жыл бұрын
Actually the colors help a great deal.
@tunneltrance
@tunneltrance 5 жыл бұрын
thank you for the knowledge
@leonardofidelcordovacastil6717
@leonardofidelcordovacastil6717 5 жыл бұрын
what happened with the sumation of all ln(n_{j}) in 11:16 instant?
@-theveniesa9508
@-theveniesa9508 3 жыл бұрын
can you please say weather, the individual systems {n(i)} can be taken as electronic energy levels in an atom
@lseinjr1
@lseinjr1 3 жыл бұрын
No, they can not. There we have to use "Fermi-Dirac" statistics instead of Boltzmann statistics.
@-theveniesa9508
@-theveniesa9508 3 жыл бұрын
@@lseinjr1 thankyou
@leonardofidelcordovacastil6717
@leonardofidelcordovacastil6717 5 жыл бұрын
many many thanks for this content
@josephmoore4764
@josephmoore4764 5 жыл бұрын
32:17 Shouldn't the sum over e^a = j*e^a or however many energy states we have?
@lseinjr1
@lseinjr1 5 жыл бұрын
In the Boltzmann distribution, all the states are non-degenerate (i.e., j=1).
@alis5893
@alis5893 4 жыл бұрын
Fantastic. Thank you
@lseinjr1
@lseinjr1 4 жыл бұрын
Glad you liked it!
@siarez
@siarez 4 жыл бұрын
From what I know about Lagrange multipliers, you can solve for the exact value of the multiplier. How come here it ends up being a variable?
@lseinjr1
@lseinjr1 4 жыл бұрын
It is because the derivation is completely general, for *any* temperature. Once you fix the temperature (T), we can solve for β= 1/kBT, so long as T not equal to 0 (which is guaranteed by the Third Law).kB is the Boltzmann constant. Does that make sense?
@siarez
@siarez 4 жыл бұрын
@@lseinjr1 My difficulty is that there should be no degree of freedom (aka undetermined variable) left after we find the optima. But here β seems to be left undetermined. In the problem setup at the beginning of the video, there is no notion of temperature in the picture. It feels like it is pulled out of the hat at the last minute.
@lseinjr1
@lseinjr1 4 жыл бұрын
In a sense, you are correct that temperature was "pulled out of the hat at the last minute." I wanted to keep the video to a manageable length, and restrict my attention to the one derivation. It would take another video of twenty (20) minutes to show exactly how we know that beta is equal to the value we said. I have such a video in progress. As a prerequisite, we need the notion of the canonical ensemble partition function for translation, which you can see here: kzbin.info/www/bejne/r17Fk3yGZtWUp6s
@siarez
@siarez 4 жыл бұрын
First, many thanks for the video. At @32:00, when we remove the sum term behind e^-α, shouldn't we multiply e^-α by the number of energy levels?
@lseinjr1
@lseinjr1 4 жыл бұрын
Great question! I should not have written the summation sign in front of the e^-a when I rewrote the exponential. Since exp (-α) is a constant, we can pull it "through" the summation sign, just as we do for integrals. Good catch!
@alexz26389
@alexz26389 3 жыл бұрын
Excellent video. At the end you rewrite \beta in terms of T. Is this a definition of temperature? If so is it possible to show it's equivalent to other definitions?
@lseinjr1
@lseinjr1 3 жыл бұрын
Yes. Perhaps the most useful definition of temperature is that, at the same temperature, two (different) gases will have the same average kinetic energy.
@asifesedov9381
@asifesedov9381 3 жыл бұрын
Why you ignored sun " ln" nj in 14:00 cuz, i 5hink you missed ln there
@lseinjr1
@lseinjr1 3 жыл бұрын
This was already noted in the Comments.
@apoorvmishra6992
@apoorvmishra6992 2 жыл бұрын
Sir I have found your video very helpful but just in the last part could you please provide some insight on using what results beta was found out to be 1/kBT?
@lseinjr1
@lseinjr1 2 жыл бұрын
That is a great question! I am working on a video to show this result theoretically, but the reasoning in it is very involved, and it will take almost as long as the entire first Boltzmann video. We can also find the result through experimentation. Suppose we are performing the NMR experiment, with just two (2) energy levels (call them E1 and E2)- with the field or against the field. If we know the field strength, we know the separation between E1 and E2. The signal strength will be proportional to the energy difference E2 - E1. Then we change the field strength, to get new E2 - E1. The signal strength will decrease as the energy difference E2 - E1 increases. If we plot several of these experiments, assuming that the state populations are Boltzmann distributed and knowing the value of the Boltzmann constant, we can then calculate beta. We could also use electronic or vibrational spectroscopy to do the same sorts of things. Does that make sense?
@apoorvmishra6992
@apoorvmishra6992 2 жыл бұрын
Thank you very much for taking time to reply. Yes it does make sense to verify the result for beta experimentally via above methods.
@paulg444
@paulg444 4 жыл бұрын
@ 13:00 what happened to the log operator on the njs ?
@lseinjr1
@lseinjr1 4 жыл бұрын
That was a typo on my part. The log operator "reappears" at 17:16. My apologies!
@chanwoo1143
@chanwoo1143 4 жыл бұрын
I must ask. How did you get the top most/first equation at 14:54? Did you use Lagrange's method of undetermined multipliers to get to this equation?? If so, then how?? This baffles me ~
@lseinjr1
@lseinjr1 4 жыл бұрын
Yes.
@chanwoo1143
@chanwoo1143 4 жыл бұрын
@@lseinjr1 How can I get the logic behind this?.. I don't get it..
@lseinjr1
@lseinjr1 4 жыл бұрын
Langrange's method is a way of solving a system of equations with additional constraints (requirements). Here, there are two (2) constraints: the first is that if we add up all the nj's, their total is N - the total number of systems in the ensemble is a constant. That is the reason for α∑nj. α is the first Lagrange multiplier The second constraint is that the total energy must be a constant. The energy of *each* system is Ejnj, and therefore, the total energy of the ensemble is a constant, ∑Ejnj. β is the second Lagrange multiplier. Does that help?
@chanwoo1143
@chanwoo1143 4 жыл бұрын
@@lseinjr1 Yes thank you very much for the explanation and for the video. That helps a lot What you have written is pretty much what is already given in the video as far as I remember. I understand that differentiating against something which is a constant will yield 0. The equation that was written at 14:54 makes sense in the mathematical sense(equation-wise?). I understand that you used this to derive boltzman distribution in the end. hmm.. I am more concerned with how you came up with this equation from the proceeding explanation given in the video. I am sorry for making my question possibly confusing. If you feel so, you don't need to answer. Anyways, again, thank you ! Best of luck!
@lseinjr1
@lseinjr1 4 жыл бұрын
The two (2) constraints were mentioned earlier in the video, when I described the characteristics of the *canonical ensemble*. (There are several different types of ensembles that are used in thermodynamics, the "microcanonical" and "grand canonical" ensembles being two (2) examples. The first constraint is mentioned at 3:41, and the second one is mentioned at 3:49. The equation at 14:39 attempts to find a relative maximum )the most probable distribution) by setting *all* first derivatives equal to 0. Since there are many nk's, each of the equations on the board i(at 14:39) is actually a *system* of equations, each relative to different variable nk. Keeping track of many variables with different indices (j and k) is certainly confusing. It can be helpful to imagine that there are just three (30 values of j and k, and to work out the derivatives using numbers rather than variables.
@mohamedessa1840
@mohamedessa1840 3 жыл бұрын
l want this prove but for degenert energe
@lseinjr1
@lseinjr1 3 жыл бұрын
The Boltzmann distribution is non-degenerate.
@arunnavinjoseph9262
@arunnavinjoseph9262 9 ай бұрын
You are changing pens too frequently.Please try to avoid that.
مسبح السرير #قصير
00:19
سكتشات وحركات
Рет қаралды 11 МЛН
Самое неинтересное видео
00:32
Miracle
Рет қаралды 1,7 МЛН
Modus males sekolah
00:14
fitrop
Рет қаралды 14 МЛН
Running With Bigger And Bigger Feastables
00:17
MrBeast
Рет қаралды 209 МЛН
Boltzmann's Entropy Equation: A History from Clausius to Planck
24:35
Kathy Loves Physics & History
Рет қаралды 89 М.
Maxwell-Boltzmann Distribution Curves (A-Level IB Chemistry)
14:27
Chemistry Student
Рет қаралды 3,2 М.
Introduction to Statistical Physics - University Physics
34:29
Pazzy Boardman
Рет қаралды 52 М.
Statistical Physics - Gibbs Entropy Derivation - University Physics
21:29
Statistical Entropy
10:37
ChemistryUTAustin
Рет қаралды 55 М.
This Is the Calculus They Won't Teach You
30:17
A Well-Rested Dog
Рет қаралды 3,2 МЛН
مسبح السرير #قصير
00:19
سكتشات وحركات
Рет қаралды 11 МЛН